-
Notifications
You must be signed in to change notification settings - Fork 1
/
chapter05.tex
453 lines (385 loc) · 21.4 KB
/
chapter05.tex
1
2
3
4
5
6
7
8
9
10
11
12
13
14
15
16
17
18
19
20
21
22
23
24
25
26
27
28
29
30
31
32
33
34
35
36
37
38
39
40
41
42
43
44
45
46
47
48
49
50
51
52
53
54
55
56
57
58
59
60
61
62
63
64
65
66
67
68
69
70
71
72
73
74
75
76
77
78
79
80
81
82
83
84
85
86
87
88
89
90
91
92
93
94
95
96
97
98
99
100
101
102
103
104
105
106
107
108
109
110
111
112
113
114
115
116
117
118
119
120
121
122
123
124
125
126
127
128
129
130
131
132
133
134
135
136
137
138
139
140
141
142
143
144
145
146
147
148
149
150
151
152
153
154
155
156
157
158
159
160
161
162
163
164
165
166
167
168
169
170
171
172
173
174
175
176
177
178
179
180
181
182
183
184
185
186
187
188
189
190
191
192
193
194
195
196
197
198
199
200
201
202
203
204
205
206
207
208
209
210
211
212
213
214
215
216
217
218
219
220
221
222
223
224
225
226
227
228
229
230
231
232
233
234
235
236
237
238
239
240
241
242
243
244
245
246
247
248
249
250
251
252
253
254
255
256
257
258
259
260
261
262
263
264
265
266
267
268
269
270
271
272
273
274
275
276
277
278
279
280
281
282
283
284
285
286
287
288
289
290
291
292
293
294
295
296
297
298
299
300
301
302
303
304
305
306
307
308
309
310
311
312
313
314
315
316
317
318
319
320
321
322
323
324
325
326
327
328
329
330
331
332
333
334
335
336
337
338
339
340
341
342
343
344
345
346
347
348
349
350
351
352
353
354
355
356
357
358
359
360
361
362
363
364
365
366
367
368
369
370
371
372
373
374
375
376
377
378
379
380
381
382
383
384
385
386
387
388
389
390
391
392
393
394
395
396
397
398
399
400
401
402
403
404
405
406
407
408
409
410
411
412
413
414
415
416
417
418
419
420
421
422
423
424
425
426
427
428
429
430
431
432
433
434
435
436
437
438
439
440
441
442
443
444
445
446
447
448
449
450
451
452
453
\chapter{连续函数空间}
\thispagestyle{empty}
\begin{exercise}
对任意 $x\in [0,1]$, 设 $f_n(x)=x^n$. 在 $[0,1]$ 上的哪些点处, $(f_n)_{n\geq 1}$ 等度连续?
\end{exercise}
\begin{solution}
因为$(f_n)_{n\geq 1}$在区间$[0,1)$上一致收敛到常值函数$f\equiv0$,
所以$(f_n)_{n\geq 1}$在$[0,1)$上等度连续,并且容易看出$(f_n)_{n\geq 1}$在$x=1$处不是等度连续的.
\end{solution}
\begin{exercise}
设 $K$ 是度量空间, $E$ 是赋范空间, $\left(f_{n}\right)_{n\geq 1}$ 是一列从 $K$ 到 $E$ 的连续函数.
证明若 $(f_{n})_{n\geq 1}$ 在一个点 $x$ 处等度连续, 则对任一收敛到 $x$
的点列 $(x_{n})_{n\geq 1}$, 都有 $(f_{n}(x)-f_{n}(x_n))_{n\geq 1}$ 收敛到 $0$.
进而证明如果 $(f_{n}(x))_{n\geq 1}$ 在 $E$ 中收敛到 $y$,
那么对任一收敛到 $x$ 的点列 $(x_{n})_{n\geq 1}$, $(f_{n}(x_{n}))_{n\geq 1}$ 也收敛到 $y$.
取 $f_{n}(x)=\sin(nx)$. 证明 $(f_{n})_{n\geq 1}$ 在 $\mathbb{R}$ 上每一点都不等度连续.
\end{exercise}
\begin{proof}
(1) 若 $(f_n)_{n\geq 1}$在点$x$处等度连续, 则对 $\forall \varepsilon>0$, $\exists \delta>0$,
使得当 $d(x,y)<\delta$ 时, 对 $\forall n\in\mathbb{N}^*$, $\|f_n(x)-f_n(y)\|<\varepsilon$.
且对任意收敛到 $x$ 的点列 $(x_n)_{n\geq 1}$, 存在正整数$N$, 当$n>N$时, $d(x_n,x)<\delta$,
故此时对任意正整数 $k$, $\|f_k(x)-f_k(x_n)\|<\varepsilon$. 取$k=n$,
得$\|f_n(x)-f_n(x_n)\|<\varepsilon$.
综上, 对$\forall \varepsilon>0$, 存在正整数 $N$, 当 $n>N$ 时, $\|f_n(x)-f_n(x_n)\|<\varepsilon$,
这就是说$(f_n(x)-f_n(x_n))_{n\geq 1}$收敛到$0$.
(2) 由于 $(f_n(x))_{n\geq 1}$ 在 $E$ 中收敛到 $y$, 故对 $\varepsilon>0$, 存在 $N_1\in\mathbb{N}^*$,
当$n>N_1$时, $\|f_n(x)-y\|<\frac{\varepsilon}{2}$, 又根据 (1),
设 $(x_n)_{n\geq 1}$ 收敛于 $x$, 则存在 $N_2\in\mathbb{N}^*$, 使得 $n>N_2$ 时,
$\|f_n(x)-f_n(x_n)\|<\frac{\varepsilon}{2}$. 取 $N=\max\left\{N_1,N_2\right\}$, 当$n>N$时,
\[\|f_n(x_n)-y\|\leq \|f_n(x_n)-f_n(x)\|+\|f_n(x)-y\|\leq\frac{\varepsilon}{2}+\frac{\varepsilon}{2}=\varepsilon.\]
故 $f_n(x_n)_{n\geq 1}$ 也收敛到 $y$.
(3) 若 $x=k\pi$, $k\in\FZ$, 则取 $x_n=k\pi+\frac{1}{n}$, 注意到$f_n(k\pi)=\sin(nk\pi)=0$,
故 $\lim\limits_{n\rightarrow\infty}f_n(k\pi)=0$, 而
\[\begin{aligned}
f_n(x_n)&=\sin(n(k\pi+\frac{1}{n}))\\&=\sin(nk\pi+1)\\
&=\sin(nk\pi)\cos1+\cos(nk\pi)\sin1\\&=\cos(nk\pi)\sin1.
\end{aligned}\]
从而 $|f_n(x_n)|=\sin 1$ 对任意正整数 $n$ 都成立,
因此 $f_n(x_n)$ 在 $n$ 趋于 $\infty$ 时极限不可能为 $0$. 由 (2) 知 $(f_n)_{n\geq 1}$ 在 $x=k\pi$ 处不等度连续.
若 $x\not= k\pi$, $k\in\FZ$, 取 $x_n=x+\frac{\pi}{n}$, 从而
\[\begin{aligned}
\|f_n(x)-f_n(x_n)\|&=|\sin (nx)-\sin(nx+\pi)|\\
&=|\sin (nx)-\sin(nx)\cos\pi-\cos(nx)\sin\pi|\\
&=2|\sin nx|.
\end{aligned}\]
下面我们说明当 $x\not=k\pi$ 时, $\lim\limits_{n\rightarrow \infty}\sin nx$不存在.
事实上, 设 $x\not=k\pi$, $k\in\FZ$, 若 $\lim\limits_{n\rightarrow \infty} \sin nx$ 存在, 那么
\[\lim\limits_{n\rightarrow \infty} (\sin((n+1)x)-\sin((n-1)x))=0.\]
由和差化积, 我们知道 $\sin((n+1)x)-\sin((n-1)x)=2\sin x\cos nx$, 从而
\[\lim\limits_{n\rightarrow \infty} \cos nx=0.\]
接着注意到 $\cos((n+1)x)=\cos nx\cos x-\sin nx\sin x$, 故
\[\lim\limits_{n\rightarrow \infty} \sin nx =0,\]
而这与 $\sin^2 nx+\cos^2 nx=1$矛盾! 从而 $\lim\limits_{n\rightarrow \infty} \sin nx$不存在.
因此当 $n$ 趋近于 $\infty$ 时, $\|f_n(x)-f_n(x_n)\|$ 极限不存在,
由 (1) 知 $(f_n)_{n\geq 1}$ 在 $x\not= k\pi$ 处不等度连续.
综上, $(f_n)_{n\geq 1}$ 在 $\mathbb{R}$上每一点都不等度连续.
\end{proof}
\begin{exercise}
设 $K$ 是拓扑空间, $(E,d)$ 是度量空间. 证明:
若 $(f_{n})$ 在 $C(K, E)$ 中依一致范数收敛, 则 $(f_{n})$ 等度连续.
\end{exercise}
\begin{proof}
设 $(f_n)_{n\geq 1}$ 一致收敛到 $f$, 容易验证 $f\in C(K,E)$,
则对 $\forall\varepsilon>0$, 存在 $N>0$, 使得当 $n>N$ 时,
$\sup_{x\in K}d(f_n(x),f(x))<\varepsilon/3$,
任取 $x\in K$, 因为 $f\in C(K,E)$, 所以存在 $V\in\mathcal{N}(x)$,
使得当 $y\in V$时, $d(f(y),f(x))<\varepsilon/3$.
因此对于上述的 $\varepsilon,N$, 当 $n>N$ 且 $y\in V$ 时, 有
\[d(f_n(y),f_n(x))\leq d(f_n(y),f(y))+d(f(y),f(x))+d(f(x),f_n(x))<\varepsilon.\]
从而集合$(f_n)_{n>N}$是等度连续的,而增加有限个元素不改变等度连续性,因此$(f_n)_{n\geq 1}$等度连续.
\end{proof}
\begin{exercise}
设 $K$ 是拓扑空间, $(E, d)$ 是度量空间, $(f_{n})$ 是 $C(K, E)$ 上等度连续序列.
证明所有使得 $(f_{n}(x))$ 是 Cauchy 序列的点 $x$ 构成的集合是 $K$ 中的闭子集.
\end{exercise}
\begin{proof}
记所有使得 $(f_n(x))$ 是 Cauchy 序列的点 $x$ 构成的集合为 $B$,
要证明 $B$ 为闭集, 只需证明其任意收敛序列的极限点仍在其中
设 $(x_k)_{k\geq 1}$ 是 $B$ 中任意一个收敛的序列, 且 $x_k\to x$.
因为$f_n\in C(K,E)$, 所以 $\forall\varepsilon>0,\exists K,\forall k>K,d(f_n(x_k),f_n(x))<\varepsilon$.
又 $x_k\in B$, 所以 $(f_n(x_k))_{n\geq 1}$ 是 Cauchy 序列,
故对于上述 $\varepsilon>0$, $\exists N$, $\forall m,n>N$,有
\[d(f_n(x_k),f_m(x_k))<\varepsilon.\]
从而
\[d(f_n(x),f_m(x))\leq d(f_n(x),f_n(x_k))+d(f_n(x_k),f_m(x_k))+d(f_m(x_k),f_m(x))<3\varepsilon.\]
这说明 $(f_n(x))_{n\geq 1}$ 是 Cauchy 序列, 故 $x\in B$, 所以 $B$ 是闭集.
\end{proof}
\begin{exercise}
考虑函数序列 $(f_{n})$, 这里 $f_{n}(t)=\sin\left(\sqrt{t+4(n\pi)^2}\right)$, $t\in[0,\infty)$.
\begin{enumerate}[(a)]
\item 证明 $(f_{n})$ 等度连续并且逐点收敛到 $0$ 函数.
\item $C_{b}([0,\infty),\mathbb{R})$ 表示 $[0, \infty)$ 上所有有界连续实函数构成的空间, 并赋予范数
\[\|f\|_{\infty}=\sup_{t\geq 0}|f(t)|.\]
$(f_n)$ 在 $C_b([0,\infty),\mathbb{R})$ 中是否相对紧?
\end{enumerate}
\end{exercise}
\begin{proof}
(a) 任意取定 $t_0\geq 0$, 对于 $\forall\varepsilon>0$,
取 $\delta=4\pi\varepsilon$, 则当 $t\in B(t_0,\delta)\cap [0,+\infty)$ 时, 对于任意的 $f_n$ 有
\begin{align*}
|f_n(t)-f_n(t_0)| & =\left|\sin(\sqrt{t+4(n\pi)^2})-\sin(\sqrt{t_0+4(n\pi)^2})\right| \\
& \leq |\sqrt{t+4(n\pi)^2}-\sqrt{t_0+4(n\pi)^2}|\\
& =\frac{|t-t_0|}{\sqrt{t+4(n\pi)^2}+\sqrt{t_0+4(n\pi)^2}}\\
& \leq\frac{|t-t_0|}{4\pi}<\varepsilon.
\end{align*}
因此 $(f_n)$ 等度连续.
对任意的 $t\in [0,\infty)$,因为
\[\begin{split}
\lim_{n\to\infty}|\sin(\sqrt{t+4(n\pi)^2})-0|&=\lim_{n\to\infty}|\sin(\sqrt{t+4(n\pi)^2})-\sin(2n\pi)|\\
&\leq \lim_{n\to\infty}\frac{t}{\sqrt{t+4(n\pi)^2}+2n\pi}\\
&\leq \lim_{n\to\infty}\frac{t}{4n\pi}=0.
\end{split}\]
故 $(f_n)$ 逐点收敛到 $0$ 函数.
(b) 注意到依范数 $\|\cdot\|_{\infty}$ 下的收敛即为在 $[0,\infty)$ 下的一致收敛,
假设 $(f_n)$有依范数 $\|\cdot\|_{\infty}$ 收敛的子列, 则由 (a) 知该子列必一致收敛于 $0$ 函数,
然而对于 $\forall n$,
\[\|f_n\|_{\infty}=\sup_{t\geq 0}|\sin(\sqrt{t+4(n\pi)^2})|=1.\]
故 $(f_n)$ 不存在依范数 $\|\cdot\|_{\infty}$ 收敛的子列, 因此 $(f_n)$ 不是相对紧的.
\end{proof}
\begin{exercise}[7]
(...)
\end{exercise}
\begin{proof}
\begin{enumerate}[(a)]
\item 对于任意 $m\geq 1$, 有 $K = \bigcup_{x\in K} B(x,1/m)$, 因为 $K$ 是紧的,
所以存在有限子集 $D_m$, 使得
\[K = \bigcup_{x\in D_m} B(x,1/m).\]
令
\[D = \bigcup_{m\geq 1} D_m.\]
显然 $D$ 是 $K$ 的可数稠密子集.
\item 因为 $D$ 可数, 故可记 $D=\{x_1,x_2,\cdots\}$.
$\{f_n(x_1):n\geq 1\}$ 相对紧, 故有收敛子列 $\{f_{n_{1k}}(x_1):k\geq 1\}$;
$\{f_n(x_2):n\geq 1\}$ 相对紧, 故 $\{f_{n_{1k}}(x_2):k\geq 1\}$有收敛子列$\{f_{n_{2k}}(x_2):k\geq 1\}$;
$\cdots$
$\{f_n(x_l):n\geq 1\}$相对紧, 故 $\{f_{n_{(l-1)k}}(x_l):k\geq 1\}$有收敛子列$\{f_{n_{lk}}(x_l):k\geq 1\}$
$\cdots$
如此进行下去,利用对角线法,可挑选出一列$(f_{n_{kk}})_{k\geq 1}$,使得对任意的$x\in D$,$(f_{n_{kk}}(x))_{k\geq 1}$收敛.
\item 对任意$x\in K$,存在$(x_m)\subset D$使得$x_m\to x$,故$\forall\varepsilon>0$,有:
\[d(f_{n_{kk}}(x),f_{n_{qq}}(x))\leq d(f_{n_{kk}}(x),f_{n_{kk}}(x_m))+d(f_{n_{kk}}(x_m),f_{n_{qq}}(x_m))+d(f_{n_{qq}}(x_m),f_{n_{qq}}(x))\]
由$(f_n)$等度连续及(b)中结论知当指标$m,k,q$都取得足够大时,有
\[d(f_{n_{kk}}(x),f_{n_{qq}}(x))<\varepsilon\]
因此 $(f_{n_{kk}}(x))$ 是 Cauchy 序列.
由 $\Delta(f_{n_{kk}},f_{n_{qq}}) = \sup_{x\in K}d((f_{n_{kk}}(x)),(f_{n_{qq}}(x)))$
知 $(f_{n_{kk}})$ 是 Cauchy 序列, 故其在$C(K,E)$中收敛. \qedhere
\end{enumerate}
\end{proof}
\begin{exercise}[10]
设 $(K,d)$ 是紧度量空间. 证明所有从 $K$ 到 $\mathbb{R}$ 的 Lipschitz 函数构成的集合在 $(C(K,\mathbb{R}),\|\cdot\|_{\infty})$ 中稠密.
\end{exercise}
\begin{proof}
记所有从 $K$ 到 $\mathbb{R}$ 的 Lipschitz 函数构成的集合为 $\mathcal{A}$.
\begin{itemize}
\item $\mathcal{A}$是 $C(K,\mathbb{R})$的子代数:
容易验证 $\mathcal{A}$ 中元素关于加法和数乘封闭,下面说明关于乘法封闭,
任意 $f,g\in\mathcal{A}$, 存在 $\lambda_1>0,\lambda_2>0$ 使得对任意 $x,y\in K$, 有
\[|f(x)-f(y)|\leq\lambda_1d(x,y).\]
\[|g(x)-g(y)|\leq\lambda_2d(x,y).\]
又因为 $K$ 为紧集, 故存在 $M_1,M_2$, 使得对于任意 $x\in K$,
有 $|f(x)|\leq M_1$, $|g(x)|\leq M_2$, 故
\[\begin{split}
|f(x)g(x)-f(y)g(y)|
& =|f(x)g(x)-f(x)g(y)+f(x)g(y)-f(y)g(y)|\\
& \leq |f(x)g(x)-f(x)g(y)|+|f(x)g(y)-f(y)g(y)|\\
& \leq M_1\lambda_2d(x,y)+M_2\lambda_1d(x,y)\\
& =(M_1\lambda_2+M_2\lambda_1)d(x,y).
\end{split}\]
从而 $\mathcal{A}$ 中元素关于乘法封闭.
\item 常值函数 $1\in\mathcal{A}$.
\item 任意取定 $y\in K$, 取 $f(x)=d(x,y)\in\mathcal{A}$, 则当 $x\neq y$ 时, $f(x)\neq f(y)$.
\end{itemize}
由 Stone-Weierstrass 定理知 $\mathcal{A}$ 在 $C(K,\mathbb{R})$ 中稠密.
\end{proof}
\begin{exercise}
设 $K_{1}$ 和 $K_{2}$ 都是紧 Hausdorff 空间.
对 $f\in C(K_{1},\FC), g\in C(K_{2},\FC)$ 定义
\[
f\otimes g(x_1,x_2)=f(x_1) g(x_2), \quad\forall(x_1,x_2)\in K_{1}\times K_{2}.
\]
并定义集合
\[
\mathcal{A}=\biggl\{\sum_{\text{有限和}} a_{i}f_{i}\otimes g_{i}: a_{i}\in\FC, f_{i}\in C(K_1,\FC), g_{i}\in C(K_2, \FC)\biggr\}.
\]
证明 $\mathcal{A}$ 在 $C(K_1\times K_2,\FC)$ 中稠密.
\end{exercise}
\begin{proof}
\begin{enumerate}[(i)]
\item $K_1\times K_2$是紧的Hausdorff空间;
\item $\mathcal{A}$ 是 $C(K_1\times K_2,\FC)$ 的子代数:容易验证$\mathcal{A}$是向量子空间,下证$\mathcal{A}$关于乘法封闭:\\
任意 $\ell_1,\ell_2\in\mathcal{A}$, 记 $\ell_1=\sum_{i\in I}a_if_{1i}\otimes g_{1i}$,
$\ell_2=\sum_{j\in J}b_jf_{2j}\otimes g_{2j}$, 其中 $I,J$ 都为有限指标集,
对任意 $(x_1,x_2)\in K_1\times K_2$, 有
\[\begin{split}
\ell_1(x_1,x_2)\cdot\ell_2(x_1,x_2)
& = \left(\sum_{i\in I}a_if_{1i}(x_1)g_{1i}(x_2)\right)
\left(\sum_{j\in J}b_jf_{2j}(x_1)g_{2j}(x_2)\right) \\
& = \sum_{i\in I}\sum_{j\in J}a_ib_j(f_{1i}f_{2j})(x_1)\cdot(g_{1i}g_{2j})(x_2) \\
& = \sum_{i\in I}\sum_{j\in J}a_ib_j\left((f_{1i}f_{2j})\otimes(g_{1i}g_{2j})\right)(x_1,x_2).
\end{split}\]
因此
\[\ell_1\cdot\ell_2
= \sum_{i\in I}\sum_{j\in J}a_ib_j\left((f_{1i}f_{2j})\otimes(g_{1i}g_{2j})\right).\]
上式仍为有限和, 结合 $C(K_1,\FC),C(K_2,\FC)$ 都为代数知 $\mathcal{A}$ 关于乘法封闭;
\item 常值函数 $1\in\mathcal{A}$
\item 可分点: 设 $x=(x_1,x_2),y=(y_1,y_2)\in K_1\times K_2$ 且$x\neq y$, 不妨设$x_1\neq y_1$,
因为 $K_1$ 是紧 Hausdorff 空间, 所以由 Urysohn 引理知存在 $f\in C(K_1,[0,1])\subset C(K_1,\FC)$
使得 $f(x_1)=0$, $f(y_1)=1$, 令 $g(x)\equiv1(\forall x\in K_2)$, 则
\[f\otimes g(x)=f\otimes g(x_1,x_2)=0,\]
\[f\otimes g(y)=f\otimes g(y_1,y_2)=1.\]
因此$\mathcal{A}$是可分点的
\item $\mathcal{A}$是自伴的
\end{enumerate}
综上知$\mathcal{A}$在$C(K_1\times K_2,\mathbb{R})$中稠密.
\end{proof}
\begin{exercise}
$[0,1]$ 上所有的偶多项式构成的集合 $\mathcal{Q}$ 是否在 $C([0,1], \mathbb{R})$ 上稠密?
$[-1,1]$ 上所有的偶多项式构成的集合 $\mathcal{R}$ 是否在 $C([-1,1], \mathbb{R})$ 上稠密?
\end{exercise}
\begin{solution}
$\mathcal{Q}$在$C([0,1],\mathbb{R})$中稠密,理由:
\begin{enumerate}[(i)]
\item $\mathcal{Q}$ 是子代数.
\item $1\in\mathcal{Q}$.
\item $\forall x,y\in [0,1]$, 取 $f(x)=x^2$, 则 $f(x)\neq f(y)$.
\end{enumerate}
但是 $\mathcal{R}$ 在 $C([-1,1],\mathbb{R})$ 中不稠密, 因为 $[-1,1]$ 中的任意一个非零点和其相反数不可分.
\end{solution}
\begin{exercise}[14]
本习题的目的是证明 Bernstein 定理: 令 $f\in C([0,1], \FK)$, 并设
\[ B_{n}(f)(x)=\sum_{k=0}^{n} \mathrm{C}_{n}^{k} f\left(\frac{k}{n}\right) x^{k}(1-x)^{n-k}.\]
则 $B_{n}$ 在 $[0,1]$ 上一致收敛到 $f$.
\begin{enumerate}[(a)]
\item 首先导出对任一正整数 $n$, 有公式
\[\sum_{k=0}^{n} \mathrm{C}_{n}^{k} k x^{k}(1-x)^{n-k}=nx \quad\text{和}\quad\sum_{k=0}^{n} \mathrm{C}_{n}^{k} k^2 x^k(1-x)^{n-k}=nx+n(n-1)x.\]
并由此证明
\[\sum_{k=0}^{n} \mathrm{C}_{n}^{k}(k-nx)^2 x^k (1-x)^{n-k}=nx(1-x).\]
\item 对任意 $\varepsilon>0$, 选择适当的 $\delta>0$, 使得
\[x, y \in[0,1],|x-y|<\delta \Rightarrow|f(x)-f(y)|<\varepsilon.\]
对任意固定的 $x\in [0,1]$, 令 $I=\{k:|x-\frac{k}{n}|<\delta\}$
及 $J=\{k:|x-\frac{k}{n}|>\delta\}$. 证明
\begin{align*}
|f(x)-B_n(f)(x)|<{}
& \varepsilon\sum_{k\in I}\mathrm{C}_n^k x^k(1-x)^{n-k} \\
& +\frac{2\|f\|_{\infty}}{\delta^2}\sum_{k\in J}\mathrm{C}_n^k \biggl(x-\frac{k}{n}\biggr)^2x^k(1-x)^{n-k}.
\end{align*}
从而导出
\[|f(x)-B_n(f)(x)|<\varepsilon+\frac{2\|f\|_{\infty}}{\delta}\frac{x(1-x)}{n}.\]
\item 得出结论
\[\lim_{n\to\infty}\|f-B_n(f)\|_{\infty}=0.\]
\end{enumerate}
\end{exercise}
\begin{proof}
(a)引入二项分布 $X\sim B(n,x)$, 则:
\[\sum_{k=0}^nC_n^kkx^k(1-x)^{n-k} = E(X) = nx.\]
\[\sum_{k=0}^nC_n^kk^2x^k(1-x)^{n-k}=E(X^2)=\operatorname{Var}(X)+E^2(X)=nx+n(n-1)x^2.\]
故
\[\sum_{k=0}^nC_n^k(k-nx)^2x^k(1-x)^{n-k}=nx+n(n-1)x^2-2nx\cdot nx+n^2x^2=nx(1-x).\]
(b)
\begin{align*}
|f(x)-B_n(f)(x)|
& = \left|f(x)-\sum_{k=0}^nC_n^kf\left(\frac{k}{n}\right)x^k(1-x)^{n-k}\right| \\
& = \left|\sum_{k=0}^nC_n^k\left(f(x)-f\left(\frac{k}{n}\right)\right)x^k(1-x)^{n-k}\right| \\
& \leq \sum_{k=0}^nC_n^k\left|f(x)-f\left(\frac{k}{n}\right)\right|x^k(1-x)^{n-k} \\
& < \varepsilon\sum_{k\in I}C_n^kx^k(1-x)^{n-k}
+ \sum_{k\in J}C_n^k\left|f(x)-f\left(\frac{k}{n}\right)\right|x^k(1-x)^{n-k} \\
& \leq \varepsilon\sum_{k\in I}C_n^kx^k(1-x)^{n-k}
+ \frac{2\|f\|_{\infty}}{\delta^2}
\sum_{k\in J}C_n^k\left(x-\frac{k}{n}\right)^2x^k(1-x)^{n-k}
\end{align*}
由 (a) 知 $\sum_{k=0}^nC_n^k(x-k/n)^2x^k(1-x)^{n-k}=x(1-x)/n$, 故
\[|f(x)-B_n(f)(x)|<\varepsilon+\frac{2\|f\|_{\infty}}{\delta^2}\frac{x(1-x)}{n}.\]
(c) 由 (b) 中所得不等式知:
\[\begin{split}
\lim_{n\to\infty}\|f-B_n(f)\|_{\infty}
& =\lim_{n\to\infty}\sup\limits_{0\leq x\leq 1}|f(x)-B_n(f)(x)| \\
& \leq \lim_{n\to\infty}\sup \limits_{0\leq x\leq 1}
\left(\varepsilon+\frac{2\|f\|_{\infty}}{\delta^2}\frac{x(1-x)}{n}\right) \\
& = \varepsilon.
\end{split}\]
由$\varepsilon$的任意性知
\[\lim_{n\to\infty}\|f-B_n(f)\|_{\infty}=0.\qedhere\]
\end{proof}
\begin{exercise}[15]
Let $f\in L_{2\pi}^1$ and define its Fourier series as
\[f \sim \sum_{n=-\infty}^\infty \widehat{f}(n) \e^{\upi nt}.\]
and the corresponding $n$-partial sum as
\[S_n(f)(t) = \sum_{k=-n}^n \widehat{f}(k) \e^{\upi kt}.\]
In this exercise, we assume that $f$ is a piecewise $C^1$ function, i.e.,
$f\in C_{2\pi}$ and there exists a partition $0<a_0<a_1<\cdots<a_k=2\pi$
such that $f$ is $C^1$ in each subinterval $(a_j,a_{j+1})$
and the limits $\lim_{x\to a_j+} f'$ and $\lim_{x\to a_{j+1}-} f'$ exist.
Prove that the Fourier series of $f$ converges to $f$ in the $\|\cdot\|_\infty$ norm and
\[\|f - S_N(f)\|_\infty \leq \sqrt{\frac{2}{N}} \|f'\|_2,\quad \forall N\geq 1.\]
\end{exercise}
\begin{exercise}
(...)
\end{exercise}
\begin{proof}
\begin{enumerate}[(a)]
\item 首先显然 $F(x)$ 是连续的, 又因为 $\hat{f}(0)=\frac{1}{2\pi}\int_0^{2\pi}f(\theta)\diff\theta=0$, 所以
\[F(x+2\pi) = \int_0^{x+2\pi}f(t)\diff t
= \int_0^xf(t)\diff t+\int_x^{x+2\pi}f(t)\diff t = \int_0^xf(t)\diff t = F(x).\]
故 $F\in C_{2\pi}$, 并且
\[\begin{split}
\hat{F}(n)
& = \frac{1}{2\pi}\int_0^{2\pi}F(\theta)e^{-in\theta}\diff\theta \\
& = \frac{1}{2\pi}\int_0^{2\pi}F(\theta)\frac{1}{-in}\diff(e^{-in\theta}) \\
& = \frac{1}{2\pi}\left(\frac{i}{n}e^{-in\theta}F(\theta)\right)
\bigg|_0^{2\pi}-\frac{1}{2\pi}\int_0^{2\pi}\frac{i}{n}e^{-in\theta}f(\theta)\diff\theta\\
& = -\frac{i}{2\pi n}\int_0^{2\pi}e^{-in\theta}f(\theta)\diff\theta \\
& = -\frac{i}{n}\hat{f}(n)
= \begin{cases}
-\frac{i}{|n|}\hat{f}(|n|)
= -\frac{i}{|n|}\frac{a_{|n|}}{2i} = -\frac{a_{|n|}}{2|n|},
& \text{if } n>0, \\
\frac{i}{|n|}\hat{f}(-|n|)=\frac{i}{|n|}\left(-\frac{a_{|n|}}{2i}\right)
= -\frac{a_{|n|}}{2|n|},
& \text{if } n<0
\end{cases} \\
& = -\frac{a_{|n|}}{2|n|}.
\end{split}\]
由上面结果知对任意 $n\geq 1$, 有
\[\begin{split}
\frac{a_n}{n}
& = -(\hat{F}(n)+\hat{F}(-n)) \\
& = -\frac{1}{2\pi}\int_0^{2\pi}F(\theta)\left(e^{-in\theta}+e^{in\theta}\right)\diff\theta\\
& = -\frac{1}{\pi}\int_0^{2\pi}F(\theta)\cos(n\theta)\diff\theta \\
& = -\frac{1}{\pi}\int_0^{2\pi}\frac{F(\theta)}{n}\diff\sin(n\theta) \\
& = -\frac{F(\theta)\sin(n\theta)}{n\pi}\bigg|_0^{2\pi}
+ \frac{1}{\pi}\int_0^{2\pi}\frac{\sin(n\theta)}{n}f(\theta)\diff\theta \\
& = \frac{1}{\pi}\int_0^{2\pi}\frac{\sin(n\theta)}{n}f(\theta)\diff\theta
\end{split}\]
故
\[\sum_{n\geq 1}\frac{a_n}{n}
= \frac{1}{\pi}\int_0^{2\pi}\sum_{n\geq 1}\frac{\sin(n\theta)}{n}f(\theta)\diff\theta.\]
由级数 $\sum_{n\geq 1}\frac{\sin(n\theta)}{n}$ 收敛及 $f\in L_{2\pi}^1$
知 $\sum_{n\geq 1}\frac{a_n}{n}$ 收敛.
\item 假设级数$\sum_{n\geq 2}\frac{\sin(nx)}{\log n}$是 Fourier 级数, 则由(a)中结论知级数
\[\sum_{n\geq 2}\frac{1}{n\log n} \text{ 收敛}.\]
显然矛盾, 因此$\sum_{n\geq 2}\frac{\sin(nx)}{\log n}$ 不是 Fourier 级数.
\end{enumerate}
\end{proof}
\begin{exercise}[17]
对任意 $a\in\mathbb{R}$, 相应的平移变换 $\tau_a(f)(x) = f(x-a)$.
证明对任意 $f\in L_{2\pi}^p$, $0<p<\infty$, 有
\[\lim_{a\to 0} \|\tau_a(f) - f\|_p = 0.\]
\end{exercise}
\begin{proof}
因为 $C_{2\pi}$ 在 $L_{2\pi}^p$ 中稠密, 所以存在函数序列$(f_n)\subset C_{2\pi}$,使得
\[\|f-f_n\|_p\to0(n\to\infty).\]
不妨设 $p\geq 1$ ($0<p<1$ 的情形同理可证), 由 Minkowski 不等式得
\[\|\tau_a(f)-f\|_p\leq\|\tau_a(f)-\tau_a(f_n)\|_p+\|\tau_a(f_n)-f_n\|_p+\|f_n-f\|_p.\]
由 $f_n$ 是一致连续函数且 $\tau_a$ 是连续变换不难知:
\[\lim_{a\to0}\|\tau_a(f)-f\|_p=0.\qedhere\]
\end{proof}
\begin{exercise}[18]
(...)
\end{exercise}
\begin{proof}
(a) (i) 由于
\[\sum_{n=0}^{N-1} \sin \biggl(n+\frac{1}{2}\biggr)t
= \biggl(\sin\frac{Nt}{2}\biggr)^2 \bigg/ \sin\frac{t}{2},\]
所以
\[F_N(t) = \frac{1}{N} \sum_{n=0}^{N-1} D_n
= \frac{1}{N} \sum_{n=0}^{N-1} \frac{\sin (n+\frac{1}{2})t}{\sin\frac{t}{2}}
= \frac{1}{N} \biggl(\frac{\sin\frac{Nt}{2}}{\sin\frac{t}{2}}\biggr)^2.\]
(ii) 由于
\[F_N(t) = \frac{1}{N} \sum_{n=0}^{N-1} \sum_{k=-n}^n \e^{\upi kx}.\]
故
\begin{align*}
\frac{1}{2\pi} \int_0^{2\pi} F_N(t) \diff t
& = \frac{1}{2\pi} \int_0^{2\pi} \frac{1}{N} \sum_{n=0}^{N-1}\sum_{k=-n}^n
\e^{\upi kt} \diff t \\
& = \frac{1}{N} \sum_{n=0}^{N-1}\sum_{k=-n}^n
\biggl[\frac{1}{2\pi} \int_0^{2\pi} \e^{\upi kt} \diff t\biggr].
\end{align*}
当 $k\neq 0$ 时, $\int_0^{2\pi} \e^{\upi kt} \diff t = 0$;
当 $k = 0$ 时, $\int_0^{2\pi} \e^{\upi kt} \diff t = 2\pi$, 故
\[\frac{1}{2\pi} \int_0^{2\pi} F_N(t) \diff t
= \frac{1}{N} \sum_{n=0}^{N-1} 1 = 1.\]
(iii) 当 $\delta\leq |t| \leq\pi$ 时, $\frac{1}{\sin^2(t/2)} \leq \frac{1}{\sin^2(\delta/2)}$,
因此
\[F_N(t) \leq \frac{1}{N} \frac{1}{\sin^2(\delta/2)},\quad \delta\leq |t|\leq \pi.\]
显然当 $N\to\infty$ 时, $F_N(t)$ 一致收敛于 $0$, 于是
\[\lim_{N\to\infty} \int_{\delta\leq |t|\leq \pi} F_N(t) \diff t = 0.\]
\end{proof}